We have your learning goals covered with Spring and Summer courses available. Enroll today!

G
Topic
First Poster
Last Poster
Colored Pencils for Math Competitions
Owinner   7
N an hour ago by mdk2013
I've heard using colored pencils is really useful for geometry problems. Is this only for very hard problems, or can it be used in MATHCOUNTS/AMC 8/10? An example problem would be much appreciated.
7 replies
Owinner
Yesterday at 5:56 PM
mdk2013
an hour ago
Practice AMC 12A
freddyfazbear   39
N 2 hours ago by greenturtle3141
Practice AMC 12A

1. Find the sum of the infinite geometric series 1/2 + 7/36 + 49/648 + …
A - 18/11, B - 9/22, C - 9/11, D - 18/7, E - 9/14

2. What is the first digit after the decimal point in the square root of 420?
A - 1, B - 2, C - 3, D - 4, E - 5

3. Two circles with radiuses 47 and 96 intersect at two points A and B. Let P be the point 82% of the way from A to B. A line is drawn through P that intersects both circles twice. Let the four intersection points, from left to right be W, X, Y, and Z. Find (PW/PX)*(PY/PZ).
A - 50/5863, B - 47/96, C - 1, D - 96/47, E - 5863/50

4. What is the largest positive integer that cannot be expressed in the form 6a + 9b + 4 + 20d, where a, b, and d are positive integers?
A - 29, B - 38, C - 43, D - 76, E - 82

5. What is the absolute difference of the probabilities of getting at least 6/10 on a 10-question true or false test and at least 3/5 on a 5-question true or false test?
A - 63/1024, B - 63/512, C - 63/256, D - 63/128, E - 0

6. How many arrangements of the letters in the word “ginger” are there such that the two vowels have an even number of letters (remember 0 is even) between them (including the original “ginger”)?
A - 72, B - 108, C - 144, D - 216, E - 432

7. After opening his final exam, Jason does not know how to solve a single question. So he decides to pull out his phone and search up the answers. Doing this, Jason has a success rate of anywhere from 94-100% for any given question he uses his phone on. However, if the teacher sees his phone at any point during the test, then Jason gets a 0.5 multiplier on his final test score, as well as he must finish the rest of the test questions without his phone. (Assume Jason uses his phone on every question he does until he finishes the test or gets caught.) Every question is a 5-choice multiple choice question. Jason has a 90% chance of not being caught with his phone. What is the expected value of Jason’s test score, rounded to the nearest tenth of a percent?
A - 89.9%, B - 90.0%, C - 90.1%, D - 90.2%, E - 90.3%

8. A criminal is caught by a police officer. Due to a lack of cooperation, the officer calls in a second officer so they can start the arrest smoothly. Officer 1 takes 26:18 to arrest a criminal, and officer 2 takes 13:09 to arrest a criminal. With these two police officers working together, how long should the arrest take?
A - 4:23, B - 5:26, C - 8:46, D - 17:32, E - 19:44

9. Statistics show that people in Memphis who eat at KFC n days a week have a (1/10)(n+2) chance of liking kool-aid, and the number of people who eat at KFC n days a week is directly proportional to 8 - n (Note that n can only be an integer from 0 to 7, inclusive). A random person in Memphis is selected. Find the probability that they like kool-aid.
A - 13/30, B - 17/30, C - 19/30, D - 23/30, E - 29/30

10 (Main). PM me for problem (I copied over this problem from the 10A but just found out a “sheriff” removed it for some reason so I don’t want to take any risks)
A - 51, B - 52, C - 53, D - 54, E - 55

10 (Alternate). Suppose that on the coordinate grid, the x-axis represents economic freedom, and the y-axis represents social freedom, where -1 <= x, y <= 1 and a higher number for either coordinate represents more freedom along that particular axis. Accordingly, the points (0, 0), (1, 1), (-1, 1), (-1, -1), and (1, -1) represent democracy, anarchy, socialism, communism, and fascism, respectively. A country is classified as whichever point it is closest to. Suppose a theoretical new country is selected by picking a random point within the square bounded by anarchy, socialism, communism, and fascism as its vertices. What is the probability that it is fascist?
A - 1 - (1/4)pi, B - 1/5, C - (1/16)pi, D - 1/4, E - 1/8

11. Two congruent towers stand near each other. Both take the shape of a right rectangular prism. A plane that cuts both towers into two pieces passes through the vertical axes of symmetry of both towers and does not cross the floor or roof of either tower. Let the point that the plane crosses the axis of symmetry of the first tower be A, and the point that the plane crosses the axis of symmetry of the second tower be B. A is 81% of the way from the floor to the roof of the first tower, and B is 69% of the way from the floor to the roof of the second tower. What percent of the total mass of both towers combined is above the plane?
A - 19%, B - 25%, C - 50%, D - 75%, E - 81%

12. On an analog clock, the minute hand makes one full revolution every hour, and the hour hand makes one full revolution every 12 hours. Both hands move at a constant rate. During which of the following time periods does the minute hand pass the hour hand?
A - 7:35 - 7:36, B - 7:36 - 7:37, C - 7:37 - 7:38, D - 7:38 - 7:39, E - 7:39 - 7:40

13. How many axes of symmetry does the graph of (x^2)(y^2) = 69 have?
A - 2, B - 3, C - 4, D - 5, E - 6

14. Let f(n) be the sum of the positive integer divisors of n. Find the sum of the digits of the smallest odd positive integer n such that f(n) is greater than 2n.
A - 15, B - 18, C - 21, D - 24, E - 27

15. A basketball has a diameter of 9 inches, and the hoop has a diameter of 18 inches. Peter decides to pick up the basketball and make a throw. Given that Peter has a 1/4 chance of accidentally hitting the backboard and missing the shot, but if he doesn’t, he is guaranteed that the frontmost point of the basketball will be within 18 inches of the center of the hoop at the moment when a great circle of the basketball crosses the plane containing the rim. No part of the ball will extend behind the backboard at any point during the throw, and the rim is attached directly to the backboard. What is the probability that Peter makes a green FN?
A - 3/128, B - 3/64, C - 3/32, D - 3/16, E - 3/8

16. Martin decides to rob 6 packages of Kool-Aid from a store. At the store, they have 5 packages each of 5 different flavors of Kool-Aid. How many different combinations of Kool-Aid could Martin rob?
A - 210, B - 205, C - 195, D - 185, E - 180

17. Find the area of a cyclic quadrilateral with side lengths 6, 9, 4, and 2, rounded to the nearest integer.
A - 16, B - 19, C - 22, D - 25, E - 28

18. Find the slope of the line tangent to the graph of y = x^2 + x + 1 at the point (2, 7).
A - 2, B - 3, C - 4, D - 5, E - 6

19. Suppose that the strength of a protest is measured in “effectiveness points”. Malcolm gathers 2048 people for a protest. During the first hour of the protest, all 2048 people protest with an effectiveness of 1 point per person. At the start of each hour of the protest after the first, half of the protestors will leave, but the ones remaining will gain one effectiveness point per person. For example, that means that during the second hour, there will be 1024 people protesting at 2 effectiveness points each, during the third hour, there will be 512 people protesting at 3 effectiveness points each, and so on. The protest will conclude at the end of the twelfth hour. After the protest is over, how many effectiveness points did it earn in total?
A - 8142, B - 8155, C - 8162, D - 8169, E - 8178

20. Find the sum of all positive integers n greater than 1 and less than 16 such that (n-1)! + 1 is divisible by n.
A - 41, B - 44, C - 47, D - 50, E - 53

21. Scientific research suggests that Stokely Carmichael had an IQ of 30. Given that IQ ranges from 1 to 200, inclusive, goes in integer increments, and the chance of having an IQ of n is proportional to n if n <= 100 and to 201 - n if n >= 101, what is the sum of the numerator and denominator of the probability that a random person is smarter than Stokely Carmichael, when expressed as a common fraction in lowest terms?
A - 1927, B - 2020, C - 2025, D - 3947, E - 3952

22. In Alabama, Jim Crow laws apply to anyone who has any positive amount of Jim Crow ancestry, no matter how small the fraction, as long as it is greater than zero. In a small town in Alabama, there were initially 9 Non-Jim Crows and 3 Jim Crows. Denote this group to be the first generation. Then those 12 people would randomly get into 6 pairs and reproduce, making the second generation, consisting of 6 people. Then the process repeats for the second generation, where they get into 3 pairs. Of the 3 people in the third generation, what is the probability that exactly one of them is Non-Jim Crow?
A - 8/27, B - 1/3, C - 52/135, D - 11/27, E - 58/135

23. Goodman, Chaney, and Schwerner each start at the point (0, 0). Assume the coordinate axes are in miles. At t = 0, Goodman starts walking along the x-axis in the positive x direction at 0.6 miles per hour, Chaney starts walking along the y-axis in the positive y direction at 0.8 miles per hour, and Schwerner starts walking along the x-axis in the negative x direction at 0.4 miles per hour. However, a clan that does not like them patrols the circumference of the circle x^2 + y^2 = 1. Three knights of the clan, equally spaced apart on the circumference of the circle, walk counterclockwise along its circumference and make one revolution every hour. At t = 0, one of the knights of the clan is at (1, 0). Any of Goodman, Chaney, and Schwerner will be caught by the clan if they walk within 50 meters of one of their 3 knights. How many of the three will be caught by the clan?
A - 0, B - 1, C - 2, D - 3, E - Not enough info to determine

24.
A list of 9 positive integers consists of 100, 112, 122, 142, 152, and 160, as well as a, b, and c, with a <= b <= c. The range of the list is 70, both the mean and median are multiples of 10, and the list has a unique mode. How many ordered triples (a, b, c) are possible?
A - 1, B - 2, C - 3, D - 4, E - 5

25. What is the integer closest to the value of tan(83)? (The 83 is in degrees)
A - 2, B - 3, C - 4, D - 6, E - 8
39 replies
freddyfazbear
Friday at 6:35 AM
greenturtle3141
2 hours ago
USA Canada math camp
Bread10   40
N 2 hours ago by cowstalker
How difficult is it to get into USA Canada math camp? What should be expected from an accepted applicant in terms of the qualifying quiz, essays and other awards or math context?
40 replies
Bread10
Mar 2, 2025
cowstalker
2 hours ago
USAMO/USAJMO Swag?!
AoPSuser412   2
N 2 hours ago by Schintalpati
I wondered if those who qualified got an email from MAA and Citadel Securities that they'd be sending out shirts. I filled out the form before the deadline but haven't received the shirt or any confirmation that it is being sent. Does anybody have theirs yet?
2 replies
AoPSuser412
6 hours ago
Schintalpati
2 hours ago
2025 Math and AI 4 Girls Competition: Win Up To $1,000!!!
audio-on   14
N 3 hours ago by FIFITHECAT
Join the 2025 Math and AI 4 Girls Competition for a chance to win up to $1,000!

Hey Everyone, I'm pleased to announce the dates for the 2025 MA4G Competition are set!
Applications will open on March 22nd, 2025, and they will close on April 26th, 2025 (@ 11:59pm PST).

Applicants will have one month to fill out an application with prizes for the top 50 contestants & cash prizes for the top 20 contestants (including $1,000 for the winner!). More details below!

Eligibility:
The competition is free to enter, and open to middle school female students living in the US (5th-8th grade).
Award recipients are selected based on their aptitude, activities and aspirations in STEM.

Event dates:
Applications will open on March 22nd, 2025, and they will close on April 26th, 2025 (by 11:59pm PST)
Winners will be announced on June 28, 2025 during an online award ceremony.

Application requirements:
Complete a 12 question problem set on math and computer science/AI related topics
Write 2 short essays

Prizes:
1st place: $1,000 Cash prize
2nd place: $500 Cash prize
3rd place: $300 Cash prize
4th-10th: $100 Cash prize each
11th-20th: $50 Cash prize each
Top 50 contestants: Over $50 worth of gadgets and stationary


Many thanks to our current and past sponsors and partners: Hudson River Trading, MATHCOUNTS, Hewlett Packard Enterprise, Automation Anywhere, JP Morgan Chase, D.E. Shaw, and AI4ALL.

Math and AI 4 Girls is a nonprofit organization aiming to encourage young girls to develop an interest in math and AI by taking part in STEM competitions and activities at an early age. The organization will be hosting an inaugural Math and AI 4 Girls competition to identify talent and encourage long-term planning of academic and career goals in STEM.

Contact:
mathandAI4girls@yahoo.com

For more information on the competition:
https://www.mathandai4girls.org/math-and-ai-4-girls-competition

More information on how to register will be posted on the website. If you have any questions, please ask here!


14 replies
audio-on
Jan 26, 2025
FIFITHECAT
3 hours ago
Congrats Team USA!
MathyMathMan   131
N 3 hours ago by MathRook7817
Congratulations to the USA team for placing 1st at the 65th IMO that took place in Bath, United Kingdom.

The team members were:

Jordan Lefkowitz
Krishna Pothapragada
Jessica Wan
Alexander Wang
Qiao Zhang
Linus Tang
131 replies
MathyMathMan
Jul 21, 2024
MathRook7817
3 hours ago
2024 AIME I Problem Ranking
zhenghua   53
N 5 hours ago by finevulture
Hi, what do you guys think the real order should've been. This is what I think:
1, 2, 3, 5, 4, 6, 11, 7, 9, 15, 8, 10, 13, 12, 14.
53 replies
zhenghua
Feb 3, 2024
finevulture
5 hours ago
advice on jmo
hexuhdecimal   0
6 hours ago
hi all, i just wanted to ask a little bit about advice on math, sorry if this is a really generic posts that exists a million times but i just wanted to ask myself. so i wanna try and make jmo next year, but im not sure how i should be studying, ive always felt that my studying was inefficient and has just been spamming problems, and ive never really taken a class. i was thinking about doing mathwoot level 1 next school year and also im doing 3 awesomemath level 2 courses this summer. is there any classes that i could take from now to the end of the school year that would help? i think right now im good with easy problems but i struggle in harder aime problems. also i think some of my fundamentals are not well built, which is why im bad at amc 10. i did really bad on amc 10 this year and on aime i did poorly as well, but after aime i looked at the problems again and thought they weren't really as hard as i thought. i wanna be able to build a better “system” where i can just look at a problem and already kinda have an idea of how to approach, but i dont know how to build that system. i dont know if i should just do more problems, learn more concepts, or take classes. i also want to try to summarize problems after doing them, but im not sure how to do that most effectively. im kind of at a roadblock and i dont really know what to do next. in the past, ive just done a lot of problems and while i definitely improved, i feel like its still not the best way for me to study. to people who made jmo or are preparing for it, how do you guys train?
0 replies
hexuhdecimal
6 hours ago
0 replies
Practice AMC 10A
freddyfazbear   53
N Yesterday at 5:27 PM by AbhayAttarde01
Hey everyone!

I’m back with another practice test. Sorry this one took a while to pump out since I have been busy lately.

Post your score/distribution, favorite problems, and thoughts on the difficulty of the test down below. Hope you enjoy!


Practice AMC 10A

1. Find the sum of the infinite geometric series 1/2 + 7/36 + 49/648 + …
A - 18/11, B - 9/22, C - 9/11, D - 18/7, E - 9/14

2. What is the first digit after the decimal point in the square root of 420?
A - 1, B - 2, C - 3, D - 4, E - 5

3. Caden’s calculator is broken and two of the digits are swapped for some reason. When he entered in 9 + 10, he got 21. What is the sum of the two digits that got swapped?
A - 2, B - 3, C - 4, D - 5, E - 6

4. Two circles with radiuses 47 and 96 intersect at two points A and B. Let P be the point 82% of the way from A to B. A line is drawn through P that intersects both circles twice. Let the four intersection points, from left to right be W, X, Y, and Z. Find (PW/PX)*(PY/PZ).
A - 50/5863, B - 47/96, C - 1, D - 96/47, E - 5863/50

5. Two dice are rolled, and the two numbers shown are a and b. How many possible values of ab are there?
A - 17, B - 18, C - 19, D - 20, E - 21

6. What is the largest positive integer that cannot be expressed in the form 6a + 9b + 4c + 20d, where a, b, c, and d are positive integers?
A - 29, B - 38, C - 43, D - 76, E - 82

7. What is the absolute difference of the probabilities of getting at least 6/10 on a 10-question true or false test and at least 3/5 on a 5-question true or false test?
A - 0, B - 1/504, C - 1/252, D - 1/126, E - 1/63

8. How many arrangements of the letters in the word “ginger” are there such that the two vowels have an even number of letters (remember 0 is even) between them (including the original “ginger”)?
A - 72, B - 108, C - 144, D - 216, E - 432

9. After opening his final exam, Jason does not know how to solve a single question. So he decides to pull out his phone and search up the answers. Doing this, Jason has a success rate of anywhere from 94-100% for any given question he uses his phone on. However, if the teacher sees his phone at any point during the test, then Jason gets a 0.5 multiplier on his final test score, as well as he must finish the rest of the test questions without his phone. (Assume Jason uses his phone on every question he does until he finishes the test or gets caught.) Every question is a 5-choice multiple choice question. Jason has a 90% chance of not being caught with his phone. What is the expected value of Jason’s test score, rounded to the nearest tenth of a percent?
A - 89.9%, B - 90.0%, C - 90.1%, D - 90.2%, E - 90.3%

10. A criminal is caught by a police officer. Due to a lack of cooperation, the officer calls in a second officer so they can start the arrest smoothly. Officer 1 takes 26:18 to arrest a criminal, and officer 2 takes 13:09 to arrest a criminal. With these two police officers working together, how long should the arrest take?
A - 4:23, B - 5:26, C - 8:46, D - 17:32, E - 19:44

11. Suppose that on the coordinate grid, the x-axis represents economic freedom, and the y-axis represents social freedom, where -1 <= x, y <= 1 and a higher number for either coordinate represents more freedom along that particular axis. Accordingly, the points (0, 0), (1, 1), (-1, 1), (-1, -1), and (1, -1) represent democracy, anarchy, socialism, communism, and fascism, respectively. A country is classified as whichever point it is closest to. Suppose a theoretical new country is selected by picking a random point within the square bounded by anarchy, socialism, communism, and fascism as its vertices. What is the probability that it is fascist?
A - 1 - (1/4)pi, B - 1/5, C - (1/16)pi, D - 1/4, E - 1/8

12. Statistics show that people in Memphis who eat at KFC n days a week have a (1/10)(n+2) chance of liking kool-aid, and the number of people who eat at KFC n days a week is directly proportional to 8 - n (Note that n can only be an integer from 0 to 7, inclusive). A random person in Memphis is selected. Find the probability that they like kool-aid.
A - 13/30, B - 17/30, C - 19/30, D - 23/30, E - 29/30

13. PM me for problem (I don’t want to post it on here again because apparently a “sheriff” got rid of it)
A - 51, B - 52, C - 53, D - 54, E - 55

14. Find the number of positive integers n less than 69 such that the average of all the squares from 1^2 to n^2, inclusive, is an integer.
A - 11, B - 12, C - 23, D - 24, E - 48

15. Find the number of ordered pairs (a, b) of integers such that (a - b)^2 = 625 - 2ab.
A - 6, B - 10, C - 12, D - 16, E - 20

16. What is the 420th digit after the decimal point in the decimal expansion of 1/13?
A - 4, B - 5, C - 6, D - 7, E - 8

17. Two congruent towers stand near each other. Both take the shape of a right rectangular prism. A plane that cuts both towers into two pieces passes through the vertical axes of symmetry of both towers and does not cross the floor or roof of either tower. Let the point that the plane crosses the axis of symmetry of the first tower be A, and the point that the plane crosses the axis of symmetry of the second tower be B. A is 81% of the way from the floor to the roof of the first tower, and B is 69% of the way from the floor to the roof of the second tower. What percent of the total mass of both towers combined is above the plane?
A - 19%, B - 25%, C - 50%, D - 75%, E - 81%

18. What is the greatest number of positive integer factors an integer from 1 to 100 can have?
A - 10, B - 12, C - 14, D - 15, E - 16

19. On an analog clock, the minute hand makes one full revolution every hour, and the hour hand makes one full revolution every 12 hours. Both hands move at a constant rate. During which of the following time periods does the minute hand pass the hour hand?
A - 7:35 - 7:36, B - 7:36 - 7:37, C - 7:37 - 7:38, D - 7:38 - 7:39, E - 7:39 - 7:40

20. Find the smallest positive integer that is a leg in three different Pythagorean triples.
A - 12, B - 14, C - 15, D - 20, E - 21

21. How many axes of symmetry does the graph of (x^2)(y^2) = 69 have?
A - 2, B - 3, C - 4, D - 5, E - 6

22. Real numbers a, b, and c are chosen uniformly and at random from 0 to 3. Find the probability that a + b + c is less than 2.
A - 4/81, B - 8/81, C - 4/27, D - 8/27, E - 2/3

23. Let f(n) be the sum of the positive integer divisors of n. Find the sum of the digits of the smallest odd positive integer n such that f(n) is greater than 2n.
A - 15, B - 18, C - 21, D - 24, E - 27

24. Find the last three digits of 24^10.
A - 376, B - 576, C - 626, D - 876, E - 926

25. A basketball has a diameter of 9 inches, and the hoop has a diameter of 18 inches. Peter decides to pick up the basketball and make a throw. Given that Peter has a 1/4 chance of accidentally hitting the backboard and missing the shot, but if he doesn’t, he is guaranteed that the frontmost point of the basketball will be within 18 inches of the center of the hoop at the moment when a great circle of the basketball crosses the plane containing the rim. No part of the ball will extend behind the backboard at any point during the throw, and the rim is attached directly to the backboard. What is the probability that Peter makes a green FN?
A - 3/128, B - 3/64, C - 3/32, D - 3/16, E - 3/8
53 replies
freddyfazbear
Mar 24, 2025
AbhayAttarde01
Yesterday at 5:27 PM
usamOOK geometry
KevinYang2.71   91
N Yesterday at 5:20 PM by mathuz
Source: USAMO 2025/4, USAJMO 2025/5
Let $H$ be the orthocenter of acute triangle $ABC$, let $F$ be the foot of the altitude from $C$ to $AB$, and let $P$ be the reflection of $H$ across $BC$. Suppose that the circumcircle of triangle $AFP$ intersects line $BC$ at two distinct points $X$ and $Y$. Prove that $C$ is the midpoint of $XY$.
91 replies
KevinYang2.71
Mar 21, 2025
mathuz
Yesterday at 5:20 PM
Inspired by IMO 1984
sqing   3
N Mar 26, 2025 by sqing
Source: Own
Let $ a,b,c\geq 0 $ and $a^2+b^2+ ab +24abc\geq\frac{81}{64}$. Prove that
$$a+b+\frac{9}{5}c\geq\frac{9}{8}$$$$a+b+\frac{3}{2}c\geq \frac{9}{8}\sqrt [3]{\frac{3}{2}}-\frac{3}{16}$$$$a+b+\frac{8}{5}c\geq  \frac{9\sqrt [3]{25}-4}{20}$$Let $ a,b,c\geq 0 $ and $ a^2+b^2+ ab +18abc\geq\frac{343}{324} $. Prove that
$$a+b+\frac{6}{5}c\geq\frac{7\sqrt 7}{18}$$$$a+b+\frac{27}{25}c\geq\frac{35\sqrt [3]5-9}{50}$$
3 replies
sqing
Mar 26, 2025
sqing
Mar 26, 2025
Inspired by IMO 1984
G H J
G H BBookmark kLocked kLocked NReply
Source: Own
The post below has been deleted. Click to close.
This post has been deleted. Click here to see post.
sqing
41310 posts
#1
Y by
Let $ a,b,c\geq 0 $ and $a^2+b^2+ ab +24abc\geq\frac{81}{64}$. Prove that
$$a+b+\frac{9}{5}c\geq\frac{9}{8}$$$$a+b+\frac{3}{2}c\geq \frac{9}{8}\sqrt [3]{\frac{3}{2}}-\frac{3}{16}$$$$a+b+\frac{8}{5}c\geq  \frac{9\sqrt [3]{25}-4}{20}$$Let $ a,b,c\geq 0 $ and $ a^2+b^2+ ab +18abc\geq\frac{343}{324} $. Prove that
$$a+b+\frac{6}{5}c\geq\frac{7\sqrt 7}{18}$$$$a+b+\frac{27}{25}c\geq\frac{35\sqrt [3]5-9}{50}$$
This post has been edited 1 time. Last edited by sqing, Mar 26, 2025, 4:05 AM
Z K Y
The post below has been deleted. Click to close.
This post has been deleted. Click here to see post.
lbh_qys
450 posts
#2
Y by
sqing wrote:
Let $ a,b,c\geq 0 $ and $a^2+b^2+ ab +24abc\geq\frac{81}{64}$. Prove that
$$a+b+\frac{3}{2}c\geq \frac{9}{8}\sqrt[3]{\frac{3}{2}}-\frac{3}{16}$$

Since
\[
(a+b)^2 + ab(24c-1)\ge \frac{81}{64},
\]if \(24c-1\le 0\), then \((a+b)^2\ge \frac{81}{64}\); hence,
\[
a+b+\frac{3}{2}c\ge a+b\ge \frac{9}{8} > \frac{9}{8}\sqrt[3]{\frac{3}{2}}-\frac{3}{16}.
\]If \(24c-1>0\), then
\[
ab(24c-1)\le \frac{(a+b)^2(24c-1)}{4},
\]so that
\[
\frac{(a+b)^2(24c+3)}{4}\ge \frac{81}{64}.
\]Thus,
\[
\bigl(8(a+b)\bigr)^2(24c+3)\ge \frac{81}{64}\times 64\times 4=18^2.
\]Furthermore, by the AM-GM inequality,
\[
\bigl(8(a+b)\bigr)^2(24c+3)\le \left(\frac{8(a+b)+8(a+b)+24c+3}{3}\right)^3=\left(\frac{16}{3}\Bigl(a+b+\frac{3}{2}c\Bigr)+1\right)^3.
\]Therefore,
\[
\frac{16}{3}\Bigl(a+b+\frac{3}{2}c\Bigr)+1\ge \sqrt[3]{18^2},
\]i.e.,
\[
a+b+\frac{3}{2}c\ge \frac{9}{8}\sqrt[3]{\frac{3}{2}}-\frac{3}{16}.
\]
This post has been edited 1 time. Last edited by lbh_qys, Mar 26, 2025, 7:03 AM
Z K Y
The post below has been deleted. Click to close.
This post has been deleted. Click here to see post.
sqing
41310 posts
#3
Y by
Let $ a,b,c\geq 0 $ and $  a(b+c+ 8bc +1)\geq\frac{28}{27} $. Prove that
$$ a+b+c \geq 1$$$$   50a+57(b+ c) \geq \frac{1400}{27}$$Very very nice.Thank lbh_qys.
sqing wrote:
Let $ a,b,c\geq 0 $ and $a^2+b^2+ ab +24abc\geq\frac{81}{64}$. Prove that $a+b+\frac{7}{5}c\geq \frac{9}{8}\sqrt [3]{\frac{7}{5}}-\frac{7}{40}$
Solution of DAVROS:
$ab=0 \implies a+b\ge\frac98 \implies a+b+\frac75c\ge\frac98> \frac{9}{8}\sqrt [3]{\frac{7}{5}}-\frac{7}{40}$ so suppose $ab>0$

Let $u=a+b, v=ab, x=a+b+\frac75c$ then $x \ge u+\frac7{120} + \frac7{120v}\left(\frac{81}{64}-u^2\right)$

$x\ge u+\frac7{120} + \frac7{30u^2}\left(\frac{81}{64}-u^2\right) = u + \frac{189}{640u^2} - \frac7{40} \ge 3\sqrt[3]{\frac{189}{4\times 640}}-\frac7{40}= \frac{9}{8}\sqrt [3]{\frac{7}{5}}-\frac{7}{40}$

$x\ge  \frac{9}{8}\sqrt [3]{\frac{7}{5}}-\frac{7}{40}$ with equality at $\frac{u}2= \frac{189}{640u^2}, a=b\implies u=\frac34\sqrt[3]{\frac75}, a=b=\frac{u}2, c=\frac57(x-u)$
This post has been edited 2 times. Last edited by sqing, Mar 27, 2025, 2:55 AM
Z K Y
The post below has been deleted. Click to close.
This post has been deleted. Click here to see post.
sqing
41310 posts
#4
Y by
Let $ a,b,c\geq 0 $ and $ a^2+b^2+c +ab+10 abc\geq\frac{28}{27}$. Prove that
$$ a+b+c \geq 1$$$$ a+b+\frac{9}{10}c \geq \frac{14}{15}$$$$ a+b+\frac{11}{10}c \geq \frac{2}{3}\sqrt{\frac{7}{3}}$$
Z K Y
N Quick Reply
G
H
=
a